Đến nội dung

Hình ảnh

$MaxP=3(xy+yz+zx)-xyz$

* * * * * 1 Bình chọn

  • Please log in to reply
Chủ đề này có 7 trả lời

#1
NguyenDangHuyYTNA

NguyenDangHuyYTNA

    Binh nhất

  • Banned
  • 41 Bài viết

$Cho x,y,z>0 thoaman x^3+y^3+z^3\leq 3.MaxP=3(xy+yz+zx)-xyz$



#2
Laxus

Laxus

    Hạ sĩ

  • Thành viên
  • 80 Bài viết

Ta có: (x-y)2+(y-z)2+(z-x)2$\geq$0

$\Leftrightarrow$ 2(x2+y2+z2-xy-xz-yz)$\geq$0

$\Leftrightarrow$ x2+y2+z2-xy-xz-yz$\geq$0

$\Leftrightarrow$ (x+y+z)2$\geq$3(x+y+z)

$\Leftrightarrow$ $\frac{(x+y+z)^{2}}{3}$$\geq$xy+yz+zx

$\Leftrightarrow$ xy+yz+zx$\leq$3

Vậy maxP=3$\Leftrightarrow$x=y=z=1


tumblr_n30f6yHnrB1qi39s1o3_500.gif

 

♠ PORTGAS D.ACE  ♠


#3
lovelyDevil

lovelyDevil

    Hạ sĩ

  • Thành viên
  • 89 Bài viết

Ta có: (x-y)2+(y-z)2+(z-x)2$\geq$0

$\Leftrightarrow$ 2(x2+y2+z2-xy-xz-yz)$\geq$0

$\Leftrightarrow$ x2+y2+z2-xy-xz-yz$\geq$0

$\Leftrightarrow$ (x+y+z)2$\geq$3(x+y+z)

$\Leftrightarrow$ $\frac{(x+y+z)^{2}}{3}$$\geq$xy+yz+zx

$\Leftrightarrow$ xy+yz+zx$\leq$3

Vậy maxP=3$\Leftrightarrow$x=y=z=1

có một sự liên quan nhẹ......



#4
Laxus

Laxus

    Hạ sĩ

  • Thành viên
  • 80 Bài viết

có một sự liên quan nhẹ......

mình đọc lộn đề thông cảm nha  :icon6:  :icon6:  :icon6:  :icon6:  :icon6:  :icon6:


tumblr_n30f6yHnrB1qi39s1o3_500.gif

 

♠ PORTGAS D.ACE  ♠


#5
QDV

QDV

    Trung sĩ

  • Thành viên
  • 131 Bài viết

$Cho x,y,z>0 thoaman x^3+y^3+z^3\leq 3.MaxP=3(xy+yz+zx)-xyz$

$P=3(1-x)(1-y)(1-z)+3(x+y+z)-3+2xyz\leq \frac{(t)^{3}}{9}+3t-1, t=3-x+y+z, 0\leq t< 3$

Dễ dàng CM $P_{max}=8 "="\Leftrightarrow x=y=z=1$


Bài viết đã được chỉnh sửa nội dung bởi QDV: 10-11-2015 - 08:42


#6
Quoc Tuan Qbdh

Quoc Tuan Qbdh

    DragonBoy

  • Điều hành viên THCS
  • 1005 Bài viết

$Cho x,y,z>0 thoaman x^3+y^3+z^3\leq 3.MaxP=3(xy+yz+zx)-xyz$

Áp dụng BĐT $Schur$ ta có :

$x(x-z)(x-y)+y(y-z)(y-x)+z(z-x)(z-y)\geq 0$

Tương đương : $x^{3}+y^{3}+z^{3}+3xyz \geq x^{2}y+y^{2}x+x^{2}z+z^{2}x+z^{2}y+y^{2}z$

$<=>x^{3}+y^{3}+z^{3}+3+3xyz \geq (x^{2}y+y^{2}x+1)+(x^{2}z+z^{2}x+1)+(z^{2}y+y^{2}z+1) \geq 3(xy+yz+zx)(AM-GM)$

$<=>x^{3}+y^{3}+z^{3}+3+2xyz \geq 3(xy+yz+zx)-xyz$

Lại có :

$3+2 \geq (x^{3}+y^{3}+z^{3})+\frac{2(x^{3}+y^{3}+z^{3})}{3} \geq x^{3}+y^{3}+z^{3}+2xyz(AM-GM)$

Suy ra : $5+3 \geq x^{3}+y^{3}+z^{3}+3+2xyz \geq 3(xy+yz+zx)-xyz$

$Max$ $P=8$ khi $x=y=z=1$ 


Bài viết đã được chỉnh sửa nội dung bởi Quoc Tuan Qbdh: 09-11-2015 - 19:19


#7
canhhoang30011999

canhhoang30011999

    Thiếu úy

  • Thành viên
  • 634 Bài viết

$P=3(1-x)(1-y)(1-z)+3(x+y+z)-3+2xyz\leq \frac{(3-t)^{2}}{9}+3t-1, t=3-x+y+z, 0\leq t< 3$

Dễ dàng CM $P_{max}=8 "="\Leftrightarrow x=y=z=1$

1-x,1-y,1-z đã lớn hơn ko đâu bạn?



#8
meomunsociu

meomunsociu

    Trung sĩ

  • Thành viên
  • 166 Bài viết

$Cho x,y,z>0 thoaman x^3+y^3+z^3\leq 3.MaxP=3(xy+yz+zx)-xyz$

Hình như đề bài đúng ra là $x^3+y^3+z^3=3$ thì phải.

Vì x;y;z > 0 nên áp dụng BĐT Cauchy ta có:

$x^3+1+1 \geq 3x$

$y^3+1+1\geq 3y$ 

$z^3+1+1\geq 3z$

=> $x+y+z\leq 3$

Ta có : $x^3+y^3+z^3=(x+y+z)(x^2+y^2+z^2-xy-yz-zx)+3xyz$

=> $xyz=1-\frac{x+y+z}{3}(x^2+y^2+z^2-xy-yz-zx)$

=> $xyz\geq 1-(x^2+y^2+z^2-xy-yz-zx)$  (do x+y+z $\leq$ 3)

=> P $\leq$$x^2+y^2+z^2-xy-yz-zx-1+3xy+3yz+3zx$

hay P $\leq (x+y+z)^2-1 \leq 8$

Dấu ''='' xảy ra <=> x=y=z=1

Vậy ......


Bài viết đã được chỉnh sửa nội dung bởi meomunsociu: 05-12-2015 - 21:28





1 người đang xem chủ đề

0 thành viên, 1 khách, 0 thành viên ẩn danh